OB/GYN exammaster

Réussis tes devoirs et examens dès maintenant avec Quizwiz!

A 25-year-old woman brings in her menstrual calendar as part of a preconceptional counseling visit. Her cycles are regular, occurring every 30 days and lasting 3-4 days. She has mild cramping on days 1 and 2 that is easily relieved by ibuprofen or acetaminophen. On what day of her cycle is she most likely ovulating? A 12 B 14 C 16 D 18 E 20

16 C Day 1 of menses is the start of a new menstrual cycle. In normally menstruating women, the luteal phase is stable at 14 days, i.e., ovulation ordinarily occurs 14 days before the onset of the next menses. Therefore, in a woman with a very regular 30-day cycle it is most likely to occur on day 16. She may, however, become pregnant if she times intercourse for two days before (B) or after (D) the day of ovulation, due to other factors such as the duration of activity of sperm. Days 12 (A) and 20 (E) are at the margins of the period of fertility, but neither is the day of likely ovulation.

A 25-year-old woman brings in her menstrual calendar as part of a preconceptional counseling visit. Her cycles are regular, occurring every 30 days and lasting 3-4 days. She has mild cramping on days 1 and 2 that is easily relieved by ibuprofen or acetaminophen. On what day of her cycle is she most likely ovulating? A 12 B 14 C 16 D 18 E 20

16 Day 1 of menses is the start of a new menstrual cycle. In normally menstruating women, the luteal phase is stable at 14 days, i.e., ovulation ordinarily occurs 14 days before the onset of the next menses. Therefore, in a woman with a very regular 30-day cycle it is most likely to occur on day 16. She may, however, become pregnant if she times intercourse for two days before (B) or after (D) the day of ovulation, due to other factors such as the duration of activity of sperm. Days 12 (A) and 20 (E) are at the margins of the period of fertility, but neither is the day of likely ovulation.

A 14-year-old girl presents because she has not yet had her 1st menstrual period; she has no other health problems. Her mother states that her daughter suffers from what seems to be mild mental retardation and attends special education classes. On examination, the girl is 4 feet 5 inches tall, her neck skin shows web-like folds on the side, her breasts are not fully developed, and the nipples are widely separated. No heart murmurs are detected; the lower limb pulse is weaker than that of the upper extremities. What is the most likely karyotype of this girl? Answer Choices 1 45,X 2 47,XYY 3 47,XXY 4 46,XY 5 47,XXX

45, X Explanation Turner syndrome is a leading cause of gonadal (ovarian) dysgenesis. It is caused by sex chromosomal abnormalities resulting in a girl/woman having only 1 X chromosome. The typical karyotype of Turner syndrome is 45,X, but it may also present with mosaic pattern 46,XX, 45,X, and sometimes 46,XX where the 2nd X chromosome is nonfunctional due to an abnormality. Turner syndrome presents in 1 of 2500 live female births. It affects many systems in the body and can be diagnosed prenatally by amniocentesis. It causes short stature, webbed neck, widely separated nipples, and intellectual disabilities. Gonadal dysgenesis leads to amenorrhea; if a mosaic pattern is the case, premature ovarian failure occurs. The gonads are always streak gonads with a small number of ovarian follicles. The pattern 47,XYY is also called super male; it is characterized by intellectual disabilities and large testicles in boys/men. The pattern 47,XXY is Klinefelter syndrome, resulting in a tall boy/man with small gonads and mild intellectual disabilities. The pattern 46,XY is the normal male karyotype. The pattern 47,XXX, is also called super female; it results in tall, relatively normal girls/women who have a lower IQ than their siblings.

A 33-year-old returns to clinic for reevaluation of her dysmenorrhea. She is a G3P3 who menstruates every 27 days. She has had a tubal ligation. She is a non-smoker. She has tried Tylenol (acetaminophen) and just completed 3 months of ibuprofen 800 mg TID x 7 days, starting 2 days before her menstrual cycle. She had only mild improvement of her symptoms. What should be the next step in management of her symptoms? A Norgestimate/ethinyl estradiol B Acetaminophen/ tramodol C Acetaminophen/ codeine D Cyclobenzaprine E Paroxetine

A Norgestimate/ethinyl estradiol A Primary dysmenorrhea is associated with ovulation, so suppression of ovulation in a woman who no longer desires fertility usually improves symptoms. Narcotics are not indicated as primary treatment in the absence of pathologic disease. Combination oral contraception will suppress ovulation.

A 53-year-old woman presents to clinic complaining of amenorrhea, irritability, and hot flashes for 6 months. She would like to try hormone replacement therapy. You are counseling her about possible risks versus benefits. Which of the following is a benefit of combined estrogen progestin therapy? A A decreased risk of breast cancer B A decreased risk of myocardial infarction C A decreased risk of stroke D A decrease in somatic symptoms E A decreased rate of cognitive decline

A decrease in somatic symptoms D The WHI study showed increase chance of cardiovascular risks and breast cancer, and showed no improvement in prevention of cognitive decline.

Your patient is a 16-year-old female who weighs 300lbs and is 5 ft. 2 inches tall. She has severe facial acne vulgaris, hirsutism, and amenorrhea. Her pediatrician has sent her to your 24-hour endocrinology clinical research station, because she thinks that the patient may have polycystic ovary syndrome. Since ovulation occurs after a sequence of steps involving changing blood hormonal levels, you could follow these levels in the patient during an entire ovarian cycle by an indwelling monitoring catheter. What would you look for to rule out the disease? Answer Choices 1 A very large rise in estrogen level during secretory phase 2 A very large drop in progesterone just before ovulation 3 A very large rise in luteinizing hormone (LH) just before ovulation 4 A very large rise in LH just before ovulation with the FSH level dropping during proliferative phase 5 A very large drop in estrogen level just before ovulation

A very large rise in luteinizing hormone (LH) just before ovulation Explanation There is a very large rise in LH level just before ovulation. FSH produces a maturing effect on the follicle and ova. In addition, the follicle produces a protein, inhibin that inhibits FSH production from the gonadotrope. This protein will become important in the events leading up to ovulation. When the follicle and ova are fully matured, LH binds to LH receptors and LH sponsors ovulation. But just prior to ovulation, the LH spike occurs by the following mechanism: at normal levels of the estradiol, the gonadotrope is inhibited, but if the levels of 17b-estradiol are very high, they produce a stimulatory effect on the gonadotrope. Just prior to ovulation, the levels of estradiol rise to extremely high levels. This stimulates the gonadotrope to produce both FSH and LH, but since FSH is inhibited by inhibin, it has a small peak, whereas LH levels rise dramatically. This is called the "LH spike." This dramatic rise in LH triggers ovulation. After ovulation, the follicle changes to become the corpus luteum. Under the primary control of LH, the corpus luteum synthesizes progesterone. Stimulation of the endometrial cells through these receptors leads to vascularization and thickening of the endometrium. These are the preliminary stages of readying the endometrial lining for trophoblast implantation. If fertilization of the ovum does not occur, then a series of events leads to the involution of the corpus luteum. Since the corpus luteum is dependent on LH, it eventually involutes under decreasing levels of LH. Involution of the corpus luteum leads to the decline of estradiol and progesterone levels. This, in turn, leads to the programmed cell death of the endometrial cells lining the uterine cavity and to the process of menses. The inhibition on the gonadotrope cell is released, and the cycle repeats itself.

A 26-year-old female presents to clinic complaining of increasing headaches for one year, irritability, bloating and fluid retention, and abdominal discomfort with loose stools during her menstrual cycle. The symptoms begin a day or two before her menses, and last until the middle of her cycle. She has tried acetaminophen and ibuprofen without improvement. On physical exam she is a well-developed, well-nourished female in no acute distress. Vitals are normal, CV and lungs are normal, pelvis exam is normal, pap smear is normal, and GC and Chlamydia testing are negative. What would be the most appropriate next step? A Pelvic ultrasound B FSH, LH levels C Fluoxetine on cycle day 21-7 D Paroxetine daily E Serum HCG

C Fluoxetine on cycle day 21-7 (Prozac) C Hx and Px are key to diagnosing premenstrual syndrome. Laboratory and radiologic procedures are not useful, particularly in light of a normal exam. With failure of NSAIDS, treatment is aimed at reducing symptoms. For mild to moderate symptoms, SSRI therapy prior to and through the menstrual cycle has become a primary therapy.

A 32-year-old nulligravida woman presents with gradual onset of excessive hair growth on the face, chest, abdomen, back, and upper parts of her limbs, together with the irregular menstruation. She denies changes in weight, body shape, and voice. Her menarche started when she was 8. Her family history is non-contributing. Physical examination reveals short stature, excessive male-pattern hair growth, and slight baldness. Her BMI is 18.5. The rest of the examination, including pelvic ultrasound, is normal. Question What is the most likely diagnosis? Answer Choices 1 Adrenal tumor 2 Ovarian tumor 3 Congenital adrenal hyperplasia 4 Polycystic ovarian syndrome 5 Idiopathic

CAH Explanation This patient has male-pattern growth of body hair. It usually occurs in androgen-stimulated parts of the body (the face, chest, areolae). In women, androgens are produced in ovaries, adrenal glands, and the hair follicle. Precocious puberty, gradual onset of the hirsutism without virilization in second or third decade, and irregular menstruation in a woman with short stature will lead you to consider late onset congenital adrenal hyperplasia. Congenital adrenal hyperplasia (CAH) is one of the most common autosomal recessive disorders. It is proven by the presence of elevated first morning 17-hydrohyprogesterone because of the deficiency of the enzyme 21- hydroxylase. Absolute or partial deficiency of the enzyme can manifest early or late in life. There are early forms in female newborn with masculinization and ambiguous genitalia (the shunting of 17-hydroxyprogesterone to the production of androgens) or in some salt-wasting cases (the deficiency in production of cortisol and aldosterone). The combination is also possible. Late onset cases are most commonly mild. They usually manifest as some type of androgen excess later in life and without the aldosterone deficiency. An adrenal tumor will present with rapid onset of virilization, which is not the case in this patient. Elevated dehydroepiandrosten sulfate (DHEAS) produced by adrenal glands will confirm the diagnosis. Ovarian tumor will also present with rapid onset of virilization, probably with palpable adnexal mass. Elevated testosterone will confirm your diagnosis. Polycystic ovarian syndrome (PCOS) is the most common endocrine disorder of reproductive-age women. It typically presents with irregular vaginal bleeding, obesity, infertility, and hirsutism, and is characterized by ovarian dysfunction, disordered gonadotropin secretion, and hyperandrogenemia. Anovulatory cycles, infertility, amenorrhea, hirsutism, acne, and acanthosis nigrans are sometimes the result. Ultrasound may reveal hyperplastic endometrium because of the absence of ovulation and progesterone cyclic shedding (this may lead to the endometrial cancer) and characteristic bilateraly enlarged ovaries with thick capsules, multiple subcapsullar follicles, and hyperplasia of ovarial stroma. The levels of testosterone might be mildly elevated, but DHEAS and 17-hydrixyprogesterone are normal. Confirmation test consists of LH:FSH ratio and decreased level of sex-hormone binding protein. BMI that puts a patient in the underweight group and short stature are not parts of the clinical picture. In most cases, hirsutism is a benign, primarily cosmetic condition. When accompanied by masculinizing signs or symptoms, particularly when these arise after puberty, hirsutism may be a manifestation of a more serious underlying disorder such as an ovarian or adrenal neoplasm. Therefore, patients with clinical evidence of hyperandrogenemia, including hirsutism and acne, should have an evaluation of adrenal androgens. Before you declare the diagnosis of idiopathic hirsutism, you should exclude those disorders.

A 52-year-old woman present to her gynecologist's office with a 6-month history of hot flashes, night sweats, mood swings, and vaginal dryness that interferes with intercourse. The symptoms seem to be worsening and are now interfering with her productivity at work and with her relationships with family and friends. Her LNMP was 8 months ago; the patient notes: "after that period, they just stopped." She denies any history of tobacco use, and she drinks one glass of red wine daily. She has a history of coronary heart disease (CHD), with stent placement 2 years ago. She recently read an article about hormone replacement and would like you to prescribe this for her. Question What is true regarding hormone replacement for this patient? Answer Choices 1 She is not a candidate for hormone replacement because of her coronary artery disease. 2 Estrogen replacement alone would be recommended due to the increased risk of endometrial cancer with progesterone administration. 3 Hormone replacement will only control her hot flashes and night sweats, but will not affect her vaginal dryness. 4 Hormone replacement is contraindicated for her because of her chronic alcohol use. 5 Certain antidepressant could be an alternate therapy for treatment of her menopause related mood swings.

Certain antidepressant could be an alternate therapy for treatment of her menopause related mood swings. Explanation This patient is presenting with classic symptoms of natural menopause and could be a candidate for hormone replacement therapy based on the history information that is provided. CHD is not a contraindication for HRT. Some studies have shown a decrease in CHD in postmenopausal women on HRT. Estrogen therapy alone is contraindicated in this patient because her uterus is intact and estrogen that is unopposed by progesterone increases the risk of endometrial cancer. Hormone replacement will in fact diminish vasomotor symptoms, as well as vaginal dryness. Modest alcohol use is not a contraindication for hormone replacement. SSRIs would be an alternative therapy option for the treatment of this patient's mood swings. However, SSRIs would not result in relief of vasomotor symptoms or vaginal dryness.

A 24-year-old female, with a history of type 2 diabetes, presents with the inability to conceive after 14 months of unprotected sexual intercourse with her husband. Her vital signs are unremarkable and you calculate a BMI of 31. Physical examination reveals acne vulgaris and hirsutism. Which of the following treatment options for her infertility would be the most effective considering your suspected diagnosis? A Medroxyprogesterone acetate B Clomiphene citrate C Metformin D Spironolactone E Mini-pill (progestin only)

Clomiphene citrate B Clomiphene citrate is highly effective as the first line treatment for infertility in PCOS. It can be accompanied with metformin, weight loss, exercise, and exogenous gonadotropins when clomiphene fails. PCOS in over half of patients is accompanied with obesity, abnormalities in insulin control, metabolic syndrome, and infertility. Medroxyprogesterone acetate (A) and the mini-pill (E) are used for endometrial protection and with oral contraceptive pills. Metformin (C) will help with her diabetes. Spironolactone (D) is a diuretic, which acts as a weak androgen receptor antagonist.

A 24-year-old female, with a history of type 2 diabetes, presents with the inability to conceive after 14 months of unprotected sexual intercourse with her husband. Vital signs are as follows: Temperature 37˚ C, BP 115/85, P 65, RR 20. Her BMI is 31. Which physical examination finding would you expect to observe with your suspected diagnosis? A Undeveloped breasts B Palpable uterus on abdominal exam C Acromegaly D Coarse dark hair on the face, chest, and back E Egophony on lung exam

D Coarse dark hair on the face, chest, and back Polycystic ovarian syndrome (PCOS) affects 5-10% of reproductive age women. It is associated with hirsutism (50% of cases) (D), obesity (80% of cases), and virilization (20%), and those affected have an increased risk of diabetes mellitus, cardiovascular disease, and metabolic syndrome. PCOS patients are often infertile. A palpable ovary may be noted on thin individuals with enlarged ovaries, not the uterus (B). Undeveloped breasts are not a diagnostic indicator for PCOS (A) and acromegaly (C) is seen with Marfan's Syndrome. Egophony (E) is when there is consolidation of the lung on exam.

A 27-year-old G 0 P 0 woman has been trying to become pregnant for the past 10 months. Menarche occurred at age 13 and her cycles have "always been irregular" except during the 10-year period when she took oral contraceptives. Since stopping contraception two years ago, she and her partner of six years have used condoms consistently until they desired pregnancy. Neither partner has a history of sexually transmitted infection, and both have been mutually monogamous. On physical examination she is 64" tall and weighs 189 pounds. She has increased dark hair on her upper lip and chin and on her lower abdomen and says she has had this hair distribution as long as she can remember. Of the following, what is the most likely cause of her apparent infertility? A Androgen-secreting ovarian neoplasm B Hypothyroidism C Pituitary adenoma D Polycystic ovarian syndrome E Premature ovarian failure

D Polycystic ovarian syndrome D Patients with polycystic ovarian syndrome typically are obese, infertile, hirsute, and have menstrual disorders, making this the most likely of the diagnoses. An androgen-secreting ovarian neoplasm (A) would cause infertility, but the associated hirsutism is rapidly progressing. Hypothyroidism (B) may cause amenorrhea and weight gain and contribute to infertility, but the overall picture is more suggestive of PCOS. The hyperprolactinemia associated with pituitary adenoma (C) might also produce mild hirsutism, but other symptoms such as galactorrhea would also likely be present. Premature ovarian failure (E) is the spontaneous cessation of menses prior to age 40, and does not apply to this woman.

Your patient is a 45-year-old woman concerned about the accelerated aging of her skin. 5 years ago, she underwent a total hysterectomy with oophorectomy. Since then, she gained about 20 pounds. She also has diabetes mellitus type II. On examination, you find a slightly overweight female (body mass index 26) with skin that has lost its elasticity and has reduced water-holding capacity, increased pigmentation, and decreased vascularity. Question What are her symptoms signs of? Answer Choices 1 Elevated testosterone 2 Decreased testosterone 3 Elevated estrogen 4 Decreased estrogen 5 Insulin resistance

Decreased estrogen Explanation Lower levels of estrogen are associated with skin aging, most probably because of telomeres shortening. The effects of reduced estrogen levels cause loss of elasticity, reduced water holding capacity, thickness, increased pigmentation, decreased vascularity in the skin, and facial hair. Some of those signs are found in your patient. Your patient has no ovaria, and testosterone is primarily secreted in the ovaries (small amounts are also secreted by the adrenal glands). Too much testosterone will cause excessive oiliness and acne rather than accelerated aging of the skin. Testosterone levels in this woman are decreased after the removal of the uterus and ovaries. Symptoms related to her skin are not characteristics of decreased testosterone levels. More often in decreased testosterone there will be a loss of muscle strength and mass, accumulation of fat, especially around the abdomen, osteoporosis, vaginal dryness, etc. High levels of estrogen can be found in women who are pregnant, extremely overweight, have diabetes, or have liver disease. They are sometimes associated with acne, red flushed appearance, spider nevi, etc. Skin changes in this patient cannot be attributed to high estrogen. Insulin resistance is a condition when insulin becomes less effective at lowering blood sugars. It can lead to weight gain and estrogen hyperproduction, resulting in skin changes attributable to high estrogen levels (acne, red flushed appearance, spider nevi, etc). Skin changes in this patient cannot be attributed to the high estrogen.

A 37-year-old female presents to the office complaining of increasing heavy vaginal bleeding with her menses. CBC reveals an iron deficient anemia. Her pelvic exam is notable for a moderately enlarged uterus. A transvaginal ultrasound reveals a 3-cm leiomyoma and a 15-cm leiomyoma. The patient wishes to preserve her fertility. Appropriate management would include which of the following? A Expectant management aimed at pain reduction B Bilateral uterine artery embolization C Reduction of blood loss with combined oral contraceptives D Immediate hysterectomy E Gonadotropin-releasing hormone analog followed by myomectomy

E Gonadotropin-releasing hormone analog followed by myomectomy E GRH causes a reversible hypogonadism, which reduces tumor size, makes surgical intervention safer, and reduces bleeding. The patient wishes to preserve her fertility, making embolization and hysterectomy non-viable options. COC do not significantly reduce bleeding, and do not regress the tumor for optimal surgical removal.

Which of the following is an acceptable method for diagnosing endometriosis in a patient who describes a history of increasing pelvic pain which is cyclic in nature and increases in intensity just before menses? Answer Choices 1 Ultrasound 2 Endometrial biopsy 3 Magnetic resonance image (MRI) 4 Serum carcinoembryonic antigen (CA 125) 5 Diagnostic laparoscopy

Diagnostic laparoscopy Explanation Endometriosis refers to the presence of endometrial glands and stroma implanted in a location outside of the intrauterine cavity. There are multiple theories regarding the mechanism by which this occurs, they include: retrograde menstruation, metaplasia of coelomic epithelium, and embryonic rests or remnants. It is a progressive disease and often results in significant management problems. Interestingly, the extent of the disease often does not correlate well with the perceived degree of discomfort. Although various imaging techniques have been described as being helpful in following the progression of known pelvic disease, such as endometriosis, none are considered acceptable for the actual diagnosis of the condition. CA 125 is a cell surface antigen found in tissues derived from coelomic epithelium, including gastrointestinal and gynecologic tissues. Although it can be use to monitor the progress of a diagnosed inflammatory or malignant process, it is not diagnostic and should not be used as a screening test. The current standard of care regarding the definitive diagnosis of endometriosis is laparoscopic visualization, preferably with a confirmatory biopsy of a lesion.

You are a general practitioner in private practice and have been treating a 12-year-old boy with headache, fever and meningismus. You suspect meningitis and send him off to the nearest hospital; an analysis of cerebral spinal fluid indicates meningococcusmeningitis. All the members of your team who have come in close contact with the boy need to take rifampicin p.o. for 2 to 4 days prophylactically. One nurse is worried because she might be 1 week pregnant. What do you do? Answer Choices 1 Give her a urine pregnancy test 2 Draw blood for quantitative ßHCG 3 Give her a one-shot rifampicin 4 Give her the same rifampicin dosage as the others 5 Substitute rifampicin with tetracycline

Draw blood for quantitative ßHCG Explanation The correct response is to draw blood for quantitative βHCG. If the nurse is only 1 week pregnant, you could prescribe the same rifampicin p.o. dosage as for the others. At that time, there is no direct blood flow between the conceptus and maternal circulation; you have to draw blood for quantitative ßHCG level to make sure that, if she is pregnant, she is not further along. A urine pregnancy test usually shows a positive result about 2 weeks after the time the next menstruation is expected. Since rifampicin is not available in injectable form, there is no possibility for a one-shot. Tetracycline is not approved for prevention of meningococcus meningitis; furthermore, it is contraindicated during pregnancy.

A 51-year-old female presents to the office complaining of intermittent vaginal spotting for three months. She has a history of well-controlled hypertension. She is a nonsmoker. Her LMP was two years ago. Her family history is significant for colon cancer. Physical exam and pelvic exam were performed and unremarkable. What is the best diagnostic step in evaluating her vaginal bleeding? A Transvaginal ultrasound B Endometrial sampling C Pap smear D Pregnancy test E STD testing

Endometrial sampling B In postmenopausal women with a family history of colon cancer, there is a 30% risk of endometrial cancer. Pregnancy and STD are less likely, and do not exclude endometrial cancer. Pap smear and ultrasound are useful but can be negative. Endometrial sampling is required to confirm or rule out cancer.

A 53-year-old woman is seen by her gynecologist. She had 3 children, and she is tubectomized. She attained menopause at 48. Over the last few weeks, she has noted some vaginal bleeding; it occurs unpredictably. She has had regular pap smears that have always been normal. The last one was 11 months prior to presentation. Speculum examination and bi-manual palpation of the genitalia reveal no abnormalities. Question What is the next best step in management of the patient? Answer Choices 1 Repeat pap smear 2 Chlamydia serology 3 Laparoscopy 4 Endometrial biopsy 5 Hormonal profile

Explanation Endometrial biopsy is the correct answer. Vaginal bleeding in a post-menopausal woman mandates endometrial sampling, with a view to ruling out endometrial cancer. Histopathological evaluation of material is essential. The most common causes of post-menopausal bleeding are vaginal atrophy (60 - 80%), exogenous estrogens (15 - 25%), endometrial cancer (10%), endometrial hypertrophy (5 - 10%), as well as cervical and endometrial polyps (2 - 12%). Repeat pap smear is incorrect. The patient had one done less than a year ago, which was normal. In light of her previously normal pap smears, the next one is not due for another 3 - 5 years. Pap smears may show the occasional dysplastic cell of endometrial origin, but it is by no means an accurate means of diagnosing endometrial pathology. Chlamydia serology is incorrect. It is done for the diagnosis of pelvic discharge in sexually active women; it is not used in cases of post-menopausal bleeding Laparoscopy is incorrect. It is not useful in the workup of post menopausal bleeding. Hormonal profile is incorrect. While exogenous estrogen is a possible cause of post menopausal bleeding, it is a diagnosis made from taking a careful history.

A 23-year-old woman presents with a history of chronic pelvic pain. She attained menarche at 13, and she has had regular periods. She has been experiencing severe pain during menses; the pain has been increasing in frequency. She has never had inter-menstrual bleeding, and she regularly uses barrier contraception. However, she has refrained from intercourse for over 6 months due to the pain it causes her. Her vital signs and general physical examination appear to be normal. A vaginal exam reveals cervical motion tenderness, with reduced mobility and nodularity of the utero-sacral ligaments. Question For a definitive diagnosis, what is the diagnostic test of choice? Answer Choices 1 HSG 2 FSH and LH levels 3 Ultrasonography 4 Laparoscopy 5 Pap smear with endocervical curettage

Explanation Laparoscopy is the correct answer. Laparoscopy is considered the gold standard in diagnosis of endometriosis. Laparoscopy allows visualization of the islands of abnormally implanted endometrial tissue. However, visual inspection alone has a high false positive rate. Confirmation requires biopsy of the abnormal areas, with histopathology demonstrating endometrial epithelium, glands, stroma, or hemosiderin-laden macrophages. Hysterosalpingography is incorrect. A visualization of the uterine cavity and tubes by this technique is useful in delineating anatomic defects, such as a bicornuate uterus, as well as mechanical obstruction by adhesion or stenosis as seen post pelvic inflammatory disease. It is not of any use in endometriosis. FSH and LH levels is an incorrect response. They may be used in the workup of infertility, but they provide no information on endometriosis. Ultrasonography is incorrect. In general, imaging techniques are not particularly helpful in diagnosis. It is useful in ruling out other pelvic pathology. Pap smear with endocervical curettage is incorrect. It is mostly a screening procedure for cervical and endocervical pathology. It can help in the identification of endometrial dysplasia and malignancy, but not endometriosis.

A 32-year-old woman presents for a routine gynecological exam. She has been married for 5 years and plans to start a family with her husband in the near future. Her first menstrual period was at the age of 11. She is in a 28/4 cycle with no irregularities or discomfort. At age 18, she had an induced abortion and has used oral contraceptives since then. In the recto-vaginal exam, you palpate a mass on the left side. An ultrasound confirms a complex cystic tumor 6cm in diameter on the left ovary. There is no free fluid in the pelvis. There is no family history of any malignant tumors. What is your therapeutic approach? Answer Choices 1 Laparoscopy and cystectomy 2 Ultrasound examination after the next period 3 Hormonal treatment with progesterone 4 Pelvic exam in 6 months 5 Ovariectomy

Explanation Since this patient is pre-menopausal, with no family history and the tumor is cystic, unilateral, asymptomatic, and there is no ascites, the possibility of it being malignant is very small. Most of these cysts regress within 2 menstruation cycles; therefore, ultrasonic control after the next period is a legitimate approach. If, after 2 cycles, the cyst persists or increases in size, laparoscopy and cystectomy would be indicated. Since the patient expressed the wish for children, it is very important to try leaving the ovary intact. Hormonal treatment with progesterone has not shown any advantage in retrogression of ovarian cysts. Even with no indication of a malignancy, a control period of 6 months is too long and should always include a sonogram. An ovariectomy should only be performed if laparoscopy shows a highly suspicious result of the tumor being malignant.

A 36-year-old woman presents for fertility counseling. She and her husband have been trying to conceive for over 1 year, during which time her periods have been irregular. Her last menses was 4 months ago. Serum hCG is negative. Serum levels of LH and FSH are elevated, and serum estradiol is low. What is the most likely diagnosis? Answer Choices 1 Tubal obstruction 2 Polycystic ovarian disease 3 Endometriosis 4 Premature menopause 5 Panhypopituitarism

Explanation The correct response is premature menopause. Low levels of estradiol associated with high levels of gonadotropic hormones indicate a functioning pituitary, therefore excluding panhypopituitarism, and primary ovarian failure. Average age of menopause in the United States remains at age 52, with a perimenopausal period averaging 5 years. Tubal obstruction, commonly the result of endometriosis, will result in infertility, but it does not affect menstruation.

A 40-year-old Caucasian woman presents with excessive menstrual bleeding, as well as bleeding between her menses; these symptoms have been occurring over her last 6 cycles. She also admits to associated "pressure and fullness" in her pelvis, dysmenorrhea, urinary frequency, and generalized fatigue. She denies being pregnant and has never had children. She denies any weight loss, fever, chills, chest pain, shortness of breath, abdominal pain, early satiety, nausea, vomiting, diarrhea, changes to her urine color or odor, flank pain, hematuria, or dysuria. Her physical exam is remarkable for a "lumpy-bumpy" cobblestone sensation above the symphysis pubis with an enlarged, mobile, and irregular contour upon bimanual palpation; no adnexal masses, cervical motion tenderness, or vaginal discharge is noted. Question What medication would be most beneficial in the initial management of this patient? Answer Choices 1 Leuprorelin (Lupron) 2 Raloxifene (Evista) 3 Clomiphene (Clomid) 4 Danocrine (Danazol) 5 Estradiol (Estraderm)

Explanation This patient's history and physical exam are most consistent with a diagnosis of uterine leiomyoma (fibroid). Pharmacologic inhibition of the growth of fibroids can be achieved by suppression of the hypothalamic-pituitary-ovarian axis through the use of gonadotropin-releasing hormone agonists (GnRH analogs) that decrease estrogen production, thereby reducing fibroid size by as much as 40-60%. It is also commonly used prior to a planned hysterectomy to reduce blood loss. Raloxifene (Evista) may help reduce fibroid growth. However, whether efficacy in reducing symptoms is comparable to that of other drugs is unclear. Clomiphene (Clomid) is indicated in the treatment of infertility, as it stimulates ovulation. Danocrine (Danazol) is an androgenic agonist and can suppress fibroid growth. However, it is not as successful as GnRH analogs and is associated with a high rate of adverse, androgenic effects such as weight gain, acne, hirsutism, edema, hair loss, deepening of the voice, flushing, sweating, and vaginal dryness; it is thus often less acceptable to patients. Estrogen replacement agents are not suitable as treatments for uterine fibroids since uterine leiomyomas are hormonally responsive benign tumors of the uterus that proliferate in response to estrogen.

A 14-year-old girl had primary amenorrhea, a webbed neck, a broad, shield-like chest with widely spaced nipples, and lack of secondary sexual characteristics. A karyotype of this patient is shown in Figure G3.6. The MOST likely diagnosis is which of the following? Answer Choices 1 Turner syndrome 2 hermaphroditism 3 androgen-insensitivity syndrome 4 Down syndrome 5 Sheehan syndrome

Explanation In classic Turner syndrome (ovarian dysgenesis), there are only 45 chromosomes, 1 sex chromosome being absent (45, X). This syndrome includes bilateral ovarian agenesis (streak ovaries), infantile genitalia, short stature, short webbed neck, high arched palate, cardiovascular and renal defects (such as coarctation of the aorta and horseshoe kidney), and cubitus valgus (an outward deviation of the extended forearm).

A 16-year-old girl has had recent onset of painful menstrual periods. Age of menarche was at 12 years. Her periods were irregular for about 8-10 months, but have been fairly regular since then, with occasional mild crampy pains on the first 2 days of her period. She rates the pains now as being 8-9 on a scale of 10. Cramps will start about a half a day before the onset of her periods, worsen the following day, and then gradually subside over the next day. There is no change with bowel movements. She also describes having a headache, looser stools, mild nausea, and low back and thigh pain. Bleeding is moderate on the first 2 days. She is otherwise healthy without history of abdominal surgery. Privately she denies any sexual activity. She is an average student and has missed 1-2 days of school with each period for the past 3 months. She also denies any fever or dysuria. There is also no family history of gynecological problems. On exam she has diffuse midline lower abdominal tenderness with some mild soreness to lower back and thighs, bowel sounds are normal, and there is no rebound tenderness. An external genital exam is normal. Based on this information, which of the following is the most likely cause of this girl's dysmenorrhea? Answer Choices 1 Primary dysmenorrhea 2 Endometriosis 3 Irritable bowel syndrome 4 Imperforate hymen 5 Ovarian cysts

Explanation Primary dysmenorrhea is associated with ovulatory cycles and typically occurs at an earlier age than secondary dysmenorrhea, as 1/3 of adolescents continue to experience anovulatory cycles in the first few years after menarche due to the time required for the hypothalamic-pituitary-ovarian cycle to mature. Disability is common with a reported 14% of girls reporting missing school or work because of pain and nearly 50% of those have pain described as moderate or severe. The physiologic basis relates to cell membrane phospholipids, endomyometrial prostaglandins, and leukotrienes. Defining symptoms include crampy midline lower abdominal pain beginning with menstrual flow or a short time before. As in this patient, cramps are typically intense on the first or second day and resolve before the end of the menstrual flow. The pain may be referred to the lower back and anterior thigh. Nausea or vomiting may occur as well as near-syncope or dizziness and complaints of weakness. Breast tenderness, bloating, headache, and mood changes can also be seen. Management involves the use of NSAIDs, which reduce the production of prostaglandins. Oral contraceptives should be considered for teens that have not experienced sufficient relief of symptoms with NSAIDs. Endometriosis occurs when functioning ectopic endometrial glands and stroma are present outside the uterine cavity, causing pain, irregular bleeding, and frequently infertility, though it may also be asymptomatic. The pelvic pain often cycles with menstruation and is associated with dysmenorrhea, dyspareunia, and infertility. The pain can be a deep constant ache with bilateral patterns of distribution and radiate to the buttock and perianal region. Pain may be associated with bladder or bowel function and there can be menstrual spotting. There is usually nonspecific pelvic tenderness on exam. Unlike this patient, it occurs more frequently in first-degree relatives of women whose endometriosis has been confirmed surgically. The pain of irritable bowel syndrome in adolescents is similar to that described in adults, with paroxysmal periumbilical abdominal pain that, unlike in this patient, is relieved by defecation or associated with an irregular pattern of defecation, including a change in frequency or consistency of stool, straining or urgency, feeling of incomplete evacuation, passage of mucus, and bloating or abdominal distention. Left lower quadrant fullness over the left colon may also be present. Periods of diarrhea alternate with periods of constipation. Imperforate hymen is recognized as a membrane covering the vaginal opening seen on external exam not evident in this patient. This may be bulging or a have a bluish hue from blood retained. There is a history of cyclic abdominal pain and a midline abdominal mass. A large enough mass may cause urinary obstruction. Ovarian cysts are fluid or semisolid filled sacs that develop on or within the ovary and are typically functional and disappear on their own. These develop from follicles that fail to rupture and release an egg and instead of being reabsorbed, the fluid within the follicle persists and forms a cyst. This may cause a sudden onset of pain that is constant or intermittent, dull or sharp aching pelvic pain that occurs, unlike the pattern seen in this patient, shortly after beginning or ending a menstrual period. There may be abnormal uterine bleeding, either longer, shorter or absent, abdominal bloating or distention, or no symptoms. Large or persistent cysts may require surgical removal.

A 15-year-old girl presents due to primary amenorrhea. She was always healthy, does not take any medications, tobacco, or drugs, and denies intercourse. She has never experienced cyclic abdominal and/or pelvic pain. Her family history is negative for gynecologic or fertility problems, autoimmune diseases, and endocrinopathies; her mother's and female relatives' menarche presented in the ages from 12-14 years. Her breasts are absent. The rest of your examination is normal. The presence of her uterus is demonstrated by ultrasound. Question What should you obtain as your next step in diagnosis of this patient? Answer Choices 1 Estrogen levels 2 Folliculostimulating hormone (FSH) levels 3 Testosterone levels 4 Pregnancy test 5 Progesterone challenge test

FSH Explanation Primary amenorrhea is the absence of menstruation by the age of 14 if the patient has no secondary sexual characteristics, or by the age of 16 if the patient has secondary sexual characteristics present. The absence of breasts indicates inadequate estrogen production. The initial workup includes a pregnancy test and serum luteinizing hormone, follicle-stimulating hormone, prolactin, and thyroid-stimulating hormone levels. If history or examination suggests a hyperandrogenic state, serum free and total testosterone and dehydroepiandrosterone sulfate concentrations are useful. FSH levels will show where the lesion is. If there are no functional ovaries, FSH will be high; and if there is no FSH, ovaries will not be stimulated. The next step should be obtaining the karyotype. The lack of X chromosome, which is necessary for developing ovarial follicles, will point to the gonadal dysgenesis, and low FSH will point to hypothalamic dysfunction (normal ovaries are not stimulated to produce estrogen). Autoimmune oophoritis with anti-ovarian antibodies should be considered when previous tests are normal. Patients with autoimmune oophoritis are at risk for the development of adrenal insufficiency and other autoimmune endocrinopathies (thyroid and parathyroid, diabetes mellitus, myasthenia gravis, pernicious anemia, etc). There is no need to obtain estrogen levels; having no breasts, your patient most probably has no estrogen. The main question is why she does not have estrogen. FSH and karyotyping will help in the diagnosis.Testosterone levels should be obtained in the patient who has breasts but has no uterus in order to decide where the estrogen originates from; if testosterone levels are that of a normal female and ovaries are present, then the patient might have Mullerian agenesis (she also will have normal pubic and axillary hair and normal karyotype). If testosterone levels are that of a normal male, the estrogen source is probably the testes, as in androgen insensitivity syndrome, when a psychologically and physically female patient (with karyotype 46, XY) presents with primary amenorrhea with the lack of pubic and axillary hair and absent uterus. A pregnancy test is indicated in secondary amenorrhea. In the case of primary amenorrhea strongly suggestive for anatomical lesion or hormonal dysfunction, pregnancy test is not indicated as a first diagnostic step. Progesterone challenge test is usually performed in the case of secondary amenorrhea when other causes are excluded (pregnancy, hypothyroidism, prolactinoma, medications). Positive test is when a single dose of progesterone or 7 days of oral medroxyprogesterone causes withdrawal bleeding, as in anovulatory cycles. Negative test will demand further evaluation with estrogen-progesterone challenge (21 days of estrogen followed by 7 days of progesterone). Negative test will prompt endometrial problems (scars, adhesions) or outflow obstruction from other cause. Withdrawal bleeding will reveal inadequate estrogen levels, and your next step will be to find out why. This can be done by obtaining FSH levels.

An 18-year-old woman presents for advice regarding emergency contraception. She had sexual intercourse the day prior to presentation; she did not use contraception, and she wants to know if it is possible that she is pregnant. Her last menstrual period (LMP) began 5 days prior to presentation. She usually gets her menses once in 28 days and the period usually lasts for 5 days. The physical exam is normal. Question What hormone is responsible for the stimulation of the ovaries at this point of the menstrual period? Answer Choices 1 Estrogen 2 Follicle-stimulating hormone (FSH) 3 Inhibins 4 Luteinizing hormone (LH) 5 Progesterone

FSH Explanation The 5th day of the menstrual period corresponds to the follicular phase of the cycle, which starts from the 1st day of bleeding to the day of the LH peak. In this phase, the FSH, which is produced by the pituitary, stimulates the development of follicles in the ovaries; only 1 follicle is dominant at the end. During the 2nd part of this phase, the granulosa cells of the follicle begin to produce estrogen. Estrogen is produced by the granulosa cells of the follicle; they are very important in the follicular phase. Their level is very low at the beginning of this phase and reaches its maximal point at the middle of the cycle, just before the the LH peak. The cells are the feedback for the LH production, and in part, for the FSH production. Inhibins are also hormones produced by granulosa cells of the follicles; they are part of the negative feedback for the FSH. Inhibin B levels rise during the luteal-follicular transition, are highest during the mid follicular phase, go up again during the LH peak, and finally decrease in the late follicular phase. Inhibins A levels decrease during the late luteal phase. Ovulation occurs approximately at the middle of the menstrual cycle (day 14). At the end of the follicular phase a rise in the level of estrogens takes place, preceding the LH peak. Ovulation will occur an average of 30 hours after this peak. The 2nd 1/2 of the menstrual cycle corresponds to the luteal phase of the menstrual cycle which starts from the day of ovulation to the 1st day of menstruation. In this phase, the ovarian corpus luteum begins to produce progesterone. If there is fertilization of the ovum, the production of progesterone continues for 4 to 5 weeks until the placenta can produce a sufficient quantity of the hormone to maintain the pregnancy. If there is no fertilization, progesterone levels decrease and menstruation occurs.

A 52-year-old woman presents because her menopausal symptoms have been extremely distressing. Over the past 4 months, she has experienced severe mood swings, hot flashes, night sweats, breast tenderness, and changes in her appetite. She has never smoked; she has an occasional drink. She had an IUD that was removed at 35 years of age. There is no family history of cancer. After a prolonged discussion, a decision to start hormone replacement therapy is made. Question What is a legitimate concern with the use of an estrogen-only supplement? Answer Choices 1 Vaginal dryness and itching 2 Worsening mood swings 3 Lowering of serum HDL level 4 Higher risk of vertebral fractures 5 Higher risk of endometrial cancer

Higher risk of endometrial cancer Explanation A higher risk of endometrial cancer is the right answer. When estrogen is administered as the sole therapy without cyclical progesterone, it causes unopposed proliferation of the endometrium. Over a period of time, this increases the chance of dysplasia, leading to endometrial cancer. To prevent this, estrogen is usually administered with cyclical progesterone. Vaginal dryness and itching is incorrect. Estrogen helps to maintain the epithelial lining of the vagina and increases lubrication, thereby decreasing the symptoms of postmenopausal atrophic vaginitis. Worsening of mood swings is incorrect. Hormone replacement therapy during menopause is beneficial in mitigating mood symptoms. When used, they are prescribed in the lowest dose possible for the shortest period of time. Lowering of serum HDL level is incorrect. Estrogen does exactly the opposite; it raises HDL and lowers LDL levels. It acts as a cardioprotective agent in that respect. Higher risk of vertebral fractures is incorrect. Estrogen influences bone metabolism and prevents osteoporosis.

A 26-year-old athlete presents complaining of scant menses x 4 months. She is a G0P0, menarche was at age 13, and her menses have been mostly regular. She is a non-smoker and non-drinker, and has been trying to achieve pregnancy x 8 months. Physical exam reveals a thin, white female in no distress. Vitals are normal, BMI is 17.5, her pelvic exam is normal, and STD cultures are negative. A pregnancy test is negative in clinic. What is the most likely cause for her amenorrhea? A Idiopathic B Hypothalamic C Hypothyroid D Polycystic ovarian syndrome E Androgenic

Hypothalamic A young healthy woman with a low BMI, no other signs of virilization, and a history of normal menarche is most likely to have induced amenorrhea, due to suppression of the hypothalamic axis from low weight and fat index.

A 26-year-old athlete presents complaining of scant menses x 4 months. She is a G0P0, menarche was at age 13, and her menses have been mostly regular. She is a non-smoker and non-drinker, and has been trying to achieve pregnancy x 8 months. Physical exam reveals a thin, white female in no distress. Vitals are normal, BMI is 17.5, her pelvic exam is normal, and STD cultures are negative. A pregnancy test is negative in clinic. What is the most likely cause for her amenorrhea? A Idiopathic B Hypothalamic C Hypothyroid D Polycystic ovarian syndrome E Androgenic

Hypothalamic B A young healthy woman with a low BMI, no other signs of virilization, and a history of normal menarche is most likely to have induced amenorrhea, due to suppression of the hypothalamic axis from low weight and fat index.

A 50-year-old woman presents for her annual pelvic examination. She states her last menstrual period was over 6 months ago; the last few occurrences of menses were extremely irregular. The patient also describes having the sensation of intense heat in her face and trunk; the sensation is accompanied by sweating. She further states that these "heat episodes" have been occurring 1 or 2 times a week for the last several months. She has no other complaints at this time. She has received her annual pap and pelvic examination yearly, as well as a clinical breast exam, without any issues. During the pelvic examination, you note obvious vaginal thinning and excessive dryness; there is also apparent vaginal wall atrophy. Question Laboratory findings in this patient would include a decreased serum estradiol as well as what? Answer Choices 1 Decreased serum thyroid stimulating hormone (TSH) 2 Increased serum follicle stimulating hormone (FSH) 3 Decreased serum luteinizing hormone (LH) 4 Increased serum prolactin level 5 Decreased serum aldosterone level

Increased serum follicle stimulating hormone (FSH) Explanation This patient scenario above is most likely to be caused from menopause. In the most pure sense of explanation, menopause is a cessation of menstruation from either natural aging (usually amenorrhea for at least 6 months) or an external cause (surgical). There is usually a 1 - 3 year time period during which women will typically adjust physiologically to the diminished hormonal and menstrual actions as well as the effects this has on their body. These effects may include hot flashes, night sweats, vaginal dryness, and in the later stages, osteoporosis. The average age of menopause in western societies is around 51 years old. Laboratory findings that indicate natural menopause include elevated levels of FSH, LH, decreased levels of estradiol (estrogen), and normal levels of TSH and prolactin. Decreased levels of TSH would be seen in hyperthyroidism, which is also referred to as thyrotoxicosis. One of the many symptoms seen with hyperthyroidism includes menstrual irregularities; however, if TSH levels are corrected, the menstrual regularity will most likely return to its normal state. Serum prolactin levels will remain unchanged in naturally occurring menopause. Levels may be increased if amenorrhea is a consequence of a prolactin-secreting pituitary adenoma. Decreased levels of aldosterone in relation to amenorrhea may be found in patients who are experiencing toxemia of pregnancy, which is not likely in this patient.

A 27-year-old woman has a history of recurrent ovarian cysts; she is being treated with a combination oral contraceptive (norethindrone/ethinyl estradiol 1mg/35mcg). The norethindrone in this drug acts to suppress ovulation by what process? Answer Choices 1 Decreasing circulating sex-hormone binding globulin (SHBG) 2 Increasing gonadotropin production 3 Inhibiting release of follicle-stimulating hormone (FSH) secretion from the anterior pituitary 4 Inhibiting release of luteinizing hormone (LH) secretion from the anterior pituitary 5 Stimulating proliferation in the endometrial lining

Inhibiting release of luteinizing hormone (LH) secretion from the anterior pituitary Explanation Norethindrone is the progestin component of the oral contraceptive, which "primarily suppresses luteinizing hormone (LH) secretion (and thus prevents ovulation)."1 The LH 'surge' is responsible for triggering ovulation. The LH is released from the anterior pituitary and is suppressed in a negative feedback cycle when progestin levels are increased. The norethindrone would be expected to increase circulating sex-hormone binding globulin (SHBG), resulting in decreased free androgens in the serum.2 Both the estrogen and progestin components of this drug decrease gonadotropin (LH and FSH) production by negative feedback to the anterior pituitary. The estrogenic agents in oral contraceptives (ethinyl estradiol) suppress follicle-stimulating hormone (FSH), as well as provide stability to the endometrium and potentiation of the progestin's actions. The effect on FSH is primarily attributed to the estrogen, while the effect on LH is primarily attributed to the progestin, such as norethindrone. Estrogen stimulates proliferation in the endometrial lining. Progestins, such as norethindrone, are associated with endometrial thinning and a shift to the secretory phase.1Neither effect on the endometrium leads to ovulation suppression.

What hormone is responsible for ovulation? Answer Choices 1 Follicle stimulating hormone 2 Luteinizing hormone 3 Estrogen 4 Testosterone 5 Insulin-like growth factor

LH Explanation Luteinizing hormone is the hormone responsible for initiation of ovulation. Follicle stimulating hormone is responsible for follicle maturation. Estrogen increases the number of luteinizing hormone (LH) receptors, which also also leads to unregulation of LH. Ultimately, a rapid rise in LH (LH surge) occurs approximately 24 - 36 hours prior to ovulation. Estrogenis produced by the conversion of testosterone to estradiol by aromatase in the granulosa cells.

A 25-year-old woman, G0P0, presents to your office with 1-year history of oligomenorrhea. Her most obvious physical exam findings are hirsutism and obesity, having a calculated BMI of 30. What would you expect to find on her workup results? Answer Choices 1 LH:FSH ratio of 2:1 or greater 2 TSH less than 0.4 mIU/mL 3 Fasting glucose less than 100 mg/dl 4 Free testosterone level less than 0.6ng/mL 5 Prolactin level greater than 40ng/mL

LH:FSH ratio of 2:1 or greater Explanation The correct answer is LH:FSH ratio of 2:1 or greater as this is the most positive indicator of polycystic ovarian syndrome (1). TSH less than 0.4 mIU/mL would indicate the possibility of hyperthyroidism and therefore is incorrect. The only symptom this patient has that would correlate with hyperthyroidism is the oligomenorrhea. A fasting glucose less than 100 mg/dl would indicate insulin sensitivity and a patient suspected of having PCOS usually demonstrates insulin resistance. A free testosterone level less than 0.6ng/mL is incorrect as 25-50% of patients with PCOS have elevations in free testosterone (1). A prolactin level greater than 40ng/mL indicates hyperprolactinemia, which usually is not a finding in PCOS and is indicative of a separate disorder (1).

A 30-year-old woman comes in for evaluation of infertility. She and her husband have been having unprotected intercourse for the past year. Her menstrual cramps have become increasingly painful, and she has a severe low backache for several days before and during her menses. She complains that intercourse is painful when her husband "goes deep." Physical examination reveals multiple tender nodules of various sizes in the posterior vaginal fornix. Definitive diagnosis is best accomplished using what methodology? A Abdominal radiography B CA-125 measurement C CT scanning of the abdomen D Laparoscopy E Pelvic ultrasonography

Laparoscopy D This woman has a classic presentation for endometriosis. Definitive or final diagnosis can only be made at laparoscopy or laparotomy, allowing direct visualization of the endometrial implants. Abdominal radiography (A), computed tomography scanning (C), and ultrasonography (E) are not usually helpful in making a definitive diagnosis. CA-125 (B) may be elevated in endometriosis, but lacks specificity.

A 50-year-old woman presents for her annual pelvic examination; her previous examination took place about 18 months ago. She tells you that her last menstrual period was over 12 months ago; the last few occurrences of menses were extremely irregular. The patient also describes having multiple daily episodes of severe, intense heat in her face and trunk that is accompanied by sweating. She further states that these "heat episodes" have been occurring 4 - 6 times a day for the last 4 weeks. They have been interfering with her everyday activities, as well as her sleep. She has no other complaints at this time. She has received her annual pelvic examinations yearly as well as clinical breast exams and mammograms; there have been no significant findings. During the pelvic examination, you note obvious vaginal thinning and excessive dryness; there is also apparent vaginal wall atrophy. Question What pharmacologic intervention can be prescribed in order to assist in ameliorating the patient's symptoms? Answer Choices 1 Menopausal hormone therapy 2 Low-dose oral contraceptives 3 Phytoestrogens 4 Progestin-releasing intrauterine devices 5 Testosterone replacement therapy

Menopausal hormone therapy Explanation This patient scenario above is most likely the result of menopause. Menopause in the most pure sense of explanation is a cessation of menstruation from either natural aging (usually amenorrhea for at least 6 months) or an external cause (surgical). There is usually a 1 -3 year time period that women will typically adjust physiologically to the diminished hormonal and menstrual actions and the effects this has on their body. These effects may include hot flashes, night sweats, vaginal dryness, and in the later stages, osteoporosis. In western societies, the average age at which women achieve menopause is about 51 years old. The patient described in the scenario above is suffering from moderate-to-severe symptoms of the physiological menopausal transition. Given that this patient does not have any obvious risk factors, menopausal hormone therapy (MHT) is the best pharmacological option at this time. At one time, MHT was referred to hormone replacement therapy. Benefits of the use of MHT include reducing hot flashes, night sweats, and other related issues, such as poor sleeping and irritability. MHT will also treat vaginal symptoms related to menopause, such as vaginal dryness and discomfort. Another benefit to its use is slowing of bone loss and easing mood swings or depressive symptoms. An extremely critical component that must be remembered by the healthcare provider and the patient if MHT is started is that the use of MHT is only recommended for a short period of time and at the lowest effective dose. Low-dose oral contraceptives (or birth control pills) may be an option for menopausal symptom relief only if the patient is perimenopausal. Because this patient has not had her menstrual period in over 8 months, she is categorized as being menopausal, so this treatment would not be appropriate. Phytoestrogens are substances found in plants, such as soy and red clover extracts, that bind to estrogen receptors. They do not appear to significantly improve menopausal hot flashes, and they would not be the best option for this patient. Progestin-releasing intrauterine devices (IUDs) would not be an appropriate treatment option as this is only used for patients using MHT to reduce the incidence or dysfunctional uterine bleeding and endometrial carcinoma. Testosterone replacement therapy in women is reserved for those patients suffering from mainly hypoactive sexual desire disorder (HSDD); other signs and symptoms that may benefit from testosterone replacement therapy include hot flushes, pubic atrophy, muscle atrophy, and osteoporosis; however, this is not the correct treatment option for this patient.

A 27-year-old woman complains of years of menstrual irregularity and increasing facial and chest hair. PMH: significant for ovarian cyst and left cytectomy. She is a non-smoker and non-drinker. Labs include a negative uCG, elevated LH, and low FSH. She desires fertility and she has not responded to three cycles of clomiphene. What would be the next choice of medication that may return ovulation? A Insulin B Metformin C Dexamethasone D Spironalactone E Finasteride

Metformin B Dexamethasone, finasteride, and spironalactone all treat symptoms of hirsutism, but do not treat the underlying cause of PCO or improve fertility outcomes. PCO has an underlying insulin resistance that can be treated with oral hypoglycemics and improve sensitivity to insulin. Adding insulin does not improve the resistance.

A 36-year-old presents to the office for evaluation of painful breasts, which is worse before her period. She complains of them feeling fuller and lumpier before onset of her menses. She has tried acetaminophen and ibuprofen with minimal relief. Her symptoms resolve at the end of her menses. What is the most likely cause of the symptoms? A Normal cyclic hormone fluctuation B Abnormal cyclic hormone fluctuation C No cyclic hormone elevation D Hormone secreting tumor E Hypothalamic dysfunction

Normal cyclic hormon fluctuation A Age, cyclic history, and resolution are essential in the diagnosis of benign fibrocystic breast disease.

A 37-year-old gravida 4, para 1, whose last normal menstrual period was 6 months ago, presents with 5-month history of irregular menses. She states her menstrual period is now 10 days in length. She uses approximately 10-12 pads or tampons daily during menses. She also has unpredictable bleeding between menstrual periods. This bleeding may begin anywhere from 2-12 days after the end of her regular period, and may last for between 2-15 days. She uses between 5-10 pads or tampons a day during this time. She denies syncope but admits to headaches and occasional dizziness. She also complains of bilateral pelvic pain that is present constantly and that radiates down both thighs. The pain is crampy, and it is relieved to some degree by naproxen sodium. On exam, the patient's vital signs are within normal limits. Abdominal exam demonstrates normal bowel sounds. There is a globular and irregular pelvic mass that extends about 5 cm above the umbilicus. The mass is nontender. There is no rebound or guarding. The pelvic exam reveals scant blood in the vaginal vault, no vaginal or cervical lesions, no cervical motion tenderness, and confirms the presence of the pelvic mass. Question What is the next best step in management for this patient? Answer Choices 1 Dilation and curettage 2 Pelvic ultrasound 3 CT scan of abdomen and pelvis, with and without contrast 4 Follicle-stimulating hormone (FSH), luteinizing hormone (LH), prolactin 5 Total and free testosterone

Pelvic ultrasound Explanation The patient's age and her history of menometrorrhagia, pelvic pain, and a globular, irregular pelvic mass are strongly suggestive of leiomyomata uteri. Fibroids are the most common solid pelvic tumors in women. The best study to evaluate the pelvic mass is ultrasound. Dilation and curettage is not indicated at this time. The patient should have an endometrial biopsy performed, because she is over the age of 35 and is experiencing abnormal vaginal bleeding, though it is probably due to leiomyomata uteri. If an endometrial biopsy is unsuccessful, then hysteroscopy and dilation and curettage would be indicated. CT scan of the abdomen and pelvis, with and without contrast, is not indicated at this time. If ultrasound exam were suspicious for malignancy, a CT scan of the abdomen and pelvis would be indicated to evaluate further the mass as well as to determine the extent of any lymphadenopathy. Hormone studies (FSH, LH, prolactin, free and total testosterone) are not generally useful in the diagnosis of a pelvic mass. They may be of use in a patient with abnormal vaginal bleeding whose pelvic exam does not reveal any masses.

A 40-year-old woman presents for a consultation. She takes a multivitamin, iron, and calcium as advised. She has never smoked cigarettes and does not drink alcohol at all. After she gave birth to her fourth child about 7 years ago, she underwent bilateral tubal ligation because both she and her husband decided not to have more children. Afterwards, she started gaining weight (now her BMI is 30). In the last several months, her periods have come every 2 weeks and last about 5-7 days. Before that they were regular: every 28-30 days, with 3-5 days of bleeding. Her mother died from breast cancer at age 70. Physical examination reveals pallor and tachycardia. Pelvic examination is normal. Question What is the next best diagnostic step? Answer Choices 1 Prescribe oral contraceptive pills 2 Check FSH 3 Perform endometrial biopsy 4 Schedule mammogram 5 Check LH

Perform endometrial biopsy Explanation Your patient most probably has dysfunctional uterine bleeding (DUB). DUB is irregular uterine bleeding that occurs in the absence of pathology or medical illness. It is a diagnosis of exclusion. Laboratory studies for patients with DUB include human chorionic gonadotropin (HCG), complete blood count (CBC), Pap smear, endometrial sampling, thyroid functions and prolactin, liver functions, coagulation studies/factors, and testing for the presence of uterine fibroids or polyps and hormone assays when indicated. Most important, however, is to exclude endometrial hyperplasia or carcinoma because your patient has several risk factors: she is older than 35 years; she is obese; and she has prolonged periods of unopposed estrogen stimulation and probably chronic anovulation. Endometrial biopsy is the most important and most commonly used diagnostic test for DUB in such a patient. In DUB there is usually a hormonal imbalance (too much estrogen thickens endometrium, and progestrone causes excessive irregular bleeding). The effect of oral contraceptive pills on endometrium mimics that of an ovulatory cycle. However, before you start oral contraceptive pills, you should confirm the diagnosis of DUB, which means excluding the other reasons. Follicle-stimulating hormone (FSH) testing may be ordered when a woman's menstrual cycle has stopped or become irregular to determine if the woman has entered menopause. That is not the case in your patient. Mammography is indicated in this woman. The National Cancer Institute recommends that women age 40 or older should have screening mammograms every 1 to 2 years. However, it is not the most important test you should order at this point; instead, you should find a reason for bleeding because of all the consequences of losing blood and because of the risk of endometrial cancer. Checking luteinizing hormone (LH) will be indicated in women having difficulties getting pregnant, having irregular or heavy menstrual periods, or having symptoms of pituitary, hypothalamic, or ovarian disorders. Again, endometrial biopsy done in the office is probably more important.

A 54-year-old female presents to clinic complaining of vaginal fullness and leaking of urine for 10 years, which is progressively getting worse. She is a G4P4 and is postmenopausal. Nothing seems to improve her symptoms, and coughing or running makes them worse. On physical exam her vitals are as follows: Ht 5'4", Wt 135 lb, T 98°F, BP 130/72. Her abdomen is soft and non-tender with no masses, and her pelvic exam reveals the anterior wall to be at the level of the hymen. In addition, UA dip is negative. She is on no medication. What would the most appropriate initial management include? A Pessary B Surgery C Renal CT D Renal functions E Cystogram

Pessary A This is a stage 2 cystocele by the Baden Walker System, with urinary incontinence. It is best treated with conservative measures. If they fail, then surgery should be considered.

A 24-year-old woman, gravida 0, presents with heavy bleeding between her periods, with cycles that seem irregular and more than 40 to 60 days apart. She has never had regular monthly periods. She is also concerned that she has acne and hair growth on her face, chest, and abdomen. She would like to become pregnant. She denies any chronic medical problems, although she states that her previous physician has advised her to lose weight because she has borderline diabetes. She is 5 feet 5 inches tall and weighs 240 lbs. Her gynecologic examination is unremarkable. Question What is the most likely explanation for her abnormal uterine bleeding? Answer Choices 1 A testosterone-secreting tumor of the ovary or adrenal 2 Increased exogenous estrogen 3 Increased exogenous progesterone 4 Loss of pulsatile GnRH due to thyroid dysfunction 5 Polycystic ovary syndrome (PCOS)

Polycystic ovary syndrome (PCOS) Explanation This patient has polycystic ovary syndrome. PCOS, also known by the name Stein-Leventhal syndrome, is an endocrine condition present in 5% to 10% of women of reproductive age. It is the most common cause of infertility in women. It is referred to as polycystic because most women with the condition have a number of small cysts in the ovaries; however, it is the characteristic constellation of signs, symptoms, and biochemical aberrations, rather than the presence of the cysts themselves, that is important in establishing the diagnosis, including dysfunctional uterine bleeding (DUB) due to estrogen breakthrough bleeding, hyperandrogenism, insulin resistance, and often obesity. Each plays a role in the evolution of an oligo-ovulatory state. It most commonly affects women ages 15 - 20 and is characterized by high estrogen and androgen levels, resulting in virilization (hair growth and acne), chronic menstrual irregularities, and infertility. Although the exact mechanism is yet to be determined, it is thought to involve a disorder of the hypothalamic-pituitary axis, in which access luteinizing hormone (LH) and androgen production causes virilization and anovulation with cyst formation in the ovary. Increased endogenous estrogen associated with obesity is likely to contribute to her anovulatory state, but there is no evidence to suggest that she is exposed to exogenous estrogen. Hyperandrogenism and, thus, virilization can be seen in the presence of a testosterone-secreting tumor of the ovary or adrenal, but her long history of oligomenorrhea makes this less likely. Thyroid dysfunction may also cause anovulation through dysregulation of a feedback loop that result in increased prolactin levels. However, her long history, including hyperandrogenism (acne and excess hair growth) and obesity, make this diagnosis less likely. A similar bleeding pattern could be seen with progestin-only contraception, such as Depo-Provera (depot medroxyprogesterone acetate/DMPA, or the "mini-pill," progestin-only birth control pill), but such hormonally-related bleeding would be due to progesterone breakthrough rather than estrogen breakthrough, as seen here. This patient desires pregnancy and is not exposed to exogenous progestin. Women with PCOS are at risk for cardiovascular illness and diabetes mellitus. They are also at increased risk for endometrial hyperplasia and endometrial cancer because of long-term unopposed estrogen stimulation of the endometrium.

A 23-year-old woman presents to clinic complaining of amenorrhea for 3 months. She also complains of increasing facial hair and weight gain. On exam, PMH: menarche age 13. Physical exam reveals a well-developed, slightly obese female with a BMI of 29. Her amenorrhea can likely be improved with which therapy? A Testosterone administration B Progesterone administration C Estrogen administration D Estrogen suppression E Testosterone suppression

Progesterone administration B Progesterone administration slows GnRH pulses, thereby improving FSH secretion and follicular maturation.

A 23-year-old woman presents to clinic complaining of amenorrhea for 3 months. She also complains of increasing facial hair and weight gain. On exam, PMH: menarche age 13. Physical exam reveals a well-developed, slightly obese female with a BMI of 29. Her amenorrhea can likely be improved with which therapy? A Testosterone administration B Progesterone administration C Estrogen administration D Estrogen suppression E Testosterone suppression

Progesterone administration Progesterone administration slows GnRH pulses, thereby improving FSH secretion and follicular maturation.

A 25-year-old woman presents to discuss her options of available contraception. You review her history and note menses onset at 12 years old, duration of menses typically around 6 days in length, and it occurs every 30 days. She is G0P0, and she has no history of abnormal pap smears or diagnosed STIs. The patient is a non-smoker, single, and in a monogamous relationship with 1 partner for the past year. All other medical history is noncontributory with the exception of the patient having a deep venous thrombosis at age 19 and a pulmonary embolism at age 21. Other than anticoagulation therapy for the appropriate amount of time, no other hematological evaluation was pursued after these events. Question Given the most likely inherited diagnosis, what would be the safest form of contraception at this time? Answer Choices 1 Estrogen/progestin combination oral contraception pill 2 Progestin-only oral contraception pill 3 Etonogestrel/ethinyl estradiol vaginal ring 4 Norelgestromin/ethinyl estradiol patch-transdermal 5 Estradiol/medroxyprogesterone monthly injection

Progestin-only oral contraception pill Explanation The correct response is progestin-only oral contraception pill. Our patient above is overall healthy 25-year-old female patient. However, her history of multiple hypercoagulable events without any substantial risk factors puts her in the likely category of possessing an inherited hypercoagulable state. The most common hypercoagulable state that is inherited currently is Factor V Leiden. Despite the fact that this patient is very highly likely to have Factor V Leiden, she can still be offered a type of contraceptive method to prevent unplanned pregnancy. The common pre-thrombotic contraceptive component is estrogen. For this reason, the patient above would be safest at beginning any method of contraception that only contains progestin. There are several types of contraception that are progestin-only: progestin-only implants, a monthly injection of only progestin, levonorgestrel based intrauterine device, and the progestin-only pills, which are also referred to as the "mini-pill." The estrogen/progestin combination oral contraception pill, the etonogestrel/ethinyl estradiol vaginal ring, the norelgestromin/ethinyl estradiol patch-transdermal, and the estradiol/medroxyprogesterone monthly injection all contain a form of estrogen, which use of this should be strictly avoided in patients such as the one described in the above scenario.

A 36-year-old woman presents for her annual gynecological examination. In addition to her routine testing, she is seeking advice on contraception. She currently takes no medication and has no known medical conditions. She smokes a 1/2 pack of cigarettes daily; she drinks 1 - 3 alcoholic drinks weekly, and she is currently sexually active with 2 partners. Her past PAP smears have all been normal. She is G2P1 and is uncertain whether she wants children in the future. Question What is your recommendation for contraception at this time? Answer Choices 1 Combination estrogen-progestin pill 2 Progestin-only pill 3 Ortho Evra (ethinyl estradiol/norelgestromin) patch 4 NuvaRing (etonogestrel/ethinyl estradiol) 5 Tubal ligation

Progestin-only pill Explanation Out of the above options, the patient should receive progestin-only pills. The patient is a smoker, which is a contraindication for estrogen containing products. The patient should be informed that she needs to take progestin-only pills at the same time daily; otherwise, she needs to use a back-up form of contraception. The patient cannot receive the combination estrogen-progestin pill, the Ortho Evra (ethinyl estradiol/norelgestromin) patch, or the NuvaRing (etonogestrel/ethinyl estradiol); they all contain estrogen. The provider should not recommend tubal ligation because the patient is uncertain whether she wants children in the future; tubal ligation is considered a permanent sterilization option.

A woman brings her 13-year-old daughter to clinic, concerned about the fact that she has not yet had her first menstrual cycle. The patient is PMH negative, Social Hx negative, FMH: mothers' age of menarche was 13. Physical exam reveals the child to be a well-developed, well-nourished female, height and weight at the 50% for age and gender. Thelarche is present and has sparse pubic hair. The remainder of her physical exam is normal for age. The most appropriate course of action is? A Reevaluation in 6 months B Thyroid evaluation C Pelvic exam D Pelvic ultrasound E hCG

Reevaluation in 6 months A The child is in the expected age for puberty, and she shows signs of hormonal changes. Thelarche- onset of breast development

A 23-year-old Asian woman presents for follow-up after being seen in the emergency department. She was seen 2 months ago in the emergency department for a 3.2 cm simple ovarian cyst, which had resolved on repeat ultrasound in 6 weeks. She reports that she had severe pain with the cyst, but all of her symptoms resolved after cyst resolution. However, she had the same experience about 18 months prior to that due to another simple ovarian cyst of 3.5 cm, and her severe pain resolved after cyst resolution. She is not sexually active, and she does not use contraception. She has no chronic medical conditions, and she takes no medications. Medical records confirm her history. The physical exam is unremarkable, showing a patient of normal body habitus, without acne and hirsutism. She is very interested in preventing future recurrences of these painful ovarian cysts. Question What should be done for this patient who wishes to prevent future ovarian cysts? Answer Choices 1 Insert a levonorgestrel intrauterine system (Mirena) 2 Perform monthly ovarian ultrasound 3 Start her on a folic acid supplement 4 Start her on a moderate-dose combined oral contraceptive pill 5 Test her CA-125 levels

Start her on a moderate-dose combined oral contraceptive pill Explanation This patient's described ovarian cysts are simple and related to follicular development. Moderate-dose combined oral contraceptives have been shown to reduce ovarian cysts in population studies. Although a mechanism of oral contraceptives is suppression of ovulation, birth control pills do not treatexisting ovarian cysts. Furthermore, the lower-dose regimens on the market currently do not appear to lower the overall incidence of ovarian cysts and achieve prevention of cysts. Starting this patient on a moderate-dose combined oral contraceptive pill is reasonable, especially if indicated for prevention of pregnancy or other reasons. Performing a monthly ultrasound may identify early cysts, but doing so will not prevent them. Ultrasounds often show incidental functional cysts in the ovaries of asymptomatic women. Ultrasound would be indicated in a symptomatic patient or to follow a concerning cyst to resolution. Once resolved, this imaging is neither necessary nor cost-efficient. Insertion of a levonorgestrel intrauterine system (Mirena) is indicated for prevention of pregnancy and menorrhagia. The primary effect of the progestin, levonorgestrel, is intrauterine; therefore, ovarian function is generally unaffected. A levonorgestrel intrauterine system would not reduce ovarian cyst recurrence. Folic acid supplementation has long been recommended for women with cervical dysplasia, but few studies have actually confirmed that supplementation reverses dysplasia. Furthermore, there are neither recommendations nor evidence for folic acid supplementation preventing ovarian cysts. Testing for CA-125 should be done in women with current ovarian cancer or an ovarian mass suggestive of an ovarian cancer. This patient denies current symptoms; she is young and described simple ovarian cysts, which resolved.

A 42-year-old gravida 1, para 1 presents with a 4-month history of menorrhagia. She is having shortened menstrual cycles that are sometimes only 15 days in length, with menstrual bleeding for 5 - 6 days. She is using approximately 12 - 14 pads or tampons per day. She admits to fatigue, headaches, and occasional dizziness, but denies syncope. There is no dysmenorrhea. A thyroid-stimulating test last month was within normal limits. Abdominal and pelvic exams are normal. What statement is true regarding dysfunctional uterine bleeding? Answer Choices 1 It is seen in all age groups equally 2 Dilatation and curettage is the only treatment 3 The condition is most common after 40 years of age 4 The condition is not seen in adolescents 5 It is never associated with ovulatory cycles

The condition is most common after 40 years of age Explanation Dysfunctional uterine bleeding (DUB) is a condition of irregular uterine bleeding in a patient who does not have an anatomic uterine lesion. It is most common above the age of 40 years (50% of the cases), but it is also seen in adolescents (20%), in whom it is associated with anovulatory cycles. Anovulatory cycles are characterized by abnormal levels of estrogen and may be due to estrogen withdrawal or breakthrough. A deteriorating ovarian follicular function is responsible for anovulatory bleeding during the climacteric. Other etiologies, such as polycystic ovarian disease, fibroids, and thyroid disease need to be ruled out before making the diagnosis. An endometrial biopsy or dilation and curettage (if the patient cannot undergo an endometrial biopsy in the office) can be diagnostic, but it is not curative or even therapeutic in a patient with DUB. Medical therapy, including estrogens, progestational agents, progesterone-impregnated intrauterine devices, and combination oral contraceptives are used to treat the condition.

A 45-year-old Caucasian woman presents with a 2-month history of a lump in her right breast. She has had no children and has been on oral contraceptive pills for 20 years. Her menstrual periods began at the age of 15 and are still regular. She is a successful business woman who does not smoke cigarettes and drinks 6 cups of coffee per day. On examination, she has a nontender lump in the upper outer quadrant of her right breast. Several matted right axillary lymph nodes are also palpable. Question Given her history, what is her greatest risk factor for developing breast cancer? Answer Choices 1 Use of oral contraceptive pills 2 Menarche at 15 years of age 3 Menopause at age 45 years of age 4 Excessive caffeine consumption 5 Nulliparity

The correct response is nulliparity. The clinical features are suggestive of breast cancer. Although it occurs mainly in women, a small percentage develops in men. Breast cancer has no single cause, but multiple risk factors have been identified. These include: Family history of premenopausal breast cancer in a first-degree relative like one's mother, daughter, or sister. Having the first full-term pregnancy after the age of 30 Prior personal history of breast cancer Chest exposure to high dose of ionizing radiation Nulliparity Early menarche at or before 12 years Late menopause after 50 years Current or recent use of oral contraceptive pills (for women who stopped using oral contraceptive pills 10+ years ago, there is no apparent increase in risk).

A 62-year-old diabetic hypertensive woman is evaluated for incontinence. She is found to have a cystocele and grade 3 uterine prolapse. Her diabetes is poorly controlled; so is her hypertension. On examination, the uterus can be repositioned with firm digital pressure. In addition to being declared unfit for surgery, she is not keen on undergoing any procedure. She has no history of any urinary infections or pelvic inflammatory disease. Question What is the next best step in management? Answer Choices 1 Anticholinergic agents to reduce bladder reactivity 2 Vaginal pessary placement 3 Laparoscopic ventral suspension of the uterus 4 Topical estrogen creams 5 Daily manual replacement of uterus

Vaginal pessary placement Explanation Vaginal pessary placement is the correct answer. Vaginal pessaries are the mainstay of conservative management of uterine prolapse. Pessaries are manually placed around the cervix, which prevents the descent of the uterus. Acute pelvic inflammatory disease and recurrent vaginitis are the only real contraindications to pessary placement. Anticholinergic agents is incorrect. Anticholinergic agents are used in the management of overactive bladders and act by reducing the bladder tone. Cholinergic nerve supply is responsible for detrusor contraction; thus, anticholinergics are useful in reducing bladder tone. Laparoscopic ventral suspension of the uterus is incorrect. While the treatment of choice for symptomatic prolapse is surgery, it is contraindicated in patients that are unfit for surgery, as is this patient. Topical estrogen creams is incorrect. It does not help uterine prolapse. Estrogen creams are useful in treating vaginal dryness and atrophic vaginitis.

A 30-year-old G1P1 Caucasian woman with a desire to become pregnant. She has been having unprotected intercourse for 12 months without pregnancy. She typically has 3 - 4 menstrual cycles a year. She reports that she had similar problems becoming pregnant with her 1st child. Her prior doctor did a complete workup for her infertility and amenorrhea. Records have been sent for your review. She successfully conceived on a medication 3 years ago, but she cannot recall the name of the medications. She has never used any method of contraception. This patient would like you to treat her infertility. Question What is the drug of choice for this patient? Answer Choices 1 Clomiphene 2 Danazol 3 Leuprolide 4 Mifepristone 5 Spironolactone

clomiphene Explanation The drug of choice for this patient is clomiphene (Clomid and others), which has estrogen agonistic and antagonistic effects, which increase gonadotropin release and follicular maturation. This medication also improves the luteinizing hormone release and estradiol secretion. Spironolactone is the medication that inhibits aldosterone and dihydrotestosterone, functioning as an androgen receptor blocker. Its primary use in gynecology is for acne, hirsutism, and polycystic ovarian syndrome. Leuprolide is a medication that causes inhibition of gonadotropin release, which reduces ovarian steroidogenesis. It is used in women for the treatment of uterine fibroids and endometriosis; it treats prostate cancer in men. It produces the opposite desired effect when given to a patient with infertility. Mifepristone is a medication with progesterone and glucocorticoid antagonistic effects, which reduce endometrial development. It is used for early pregnancy termination; off-label, it is used for endometriosis, uterine fibroids, and postcoital contraception. This medication should not be given to this patient. Danazol is an older medication that suppresses follicle stimulating hormone (FSH) and luteinizing hormone (LH). It reduces estrogen production and is weakly androgenic. This medication would worsen the patient's chances of conception.

A 23-year-old complains of chronic pelvic pain. It is worse with intercourse, several days before her period, and throughout her period. She is a GOPO, LMP two weeks ago. She is married and has had one partner for the last three years. Her symptoms have been increasing over the last year. What would the most appropriate initial management of her symptoms include? A Expectant management B Combined oral contraceptives C Surgical intervention D Acetaminophen E Androgen therapy

combined oral contraceptives B Endometriosis treatment is aimed at reducing pain and preserving fertility. Surgical interventional is not first line therapy until medication has been tried. NSAIDS are used, as opposed to acetaminophen, to lower the prostaglandin levels. Combined oral contraceptives suppress ovulation, decrease menstrual flow, and decidualize implants.

A 25-year-old Caucasian woman and her husband have been trying to have a baby for the last 2 years. The patients' medical history included cyclical pelvic pain, dysmenorrhea and dyspareunia. The physical exam reveals the following: diffuse abdominal or pelvic pain of variable location, nodular thickening and tenderness along the uterosacral ligaments, on the posterior surface of the uterus, and in the posterior cul-de-sac, scarring and narrowing of the posterior vaginal fornix, and adnexal enlargement and tenderness. Question What is the most likely diagnosis? Answer Choices 1 Endometriosis 2 Ectopic pregnancy 3 Adnexal mass 4 Pelvic relaxation 5 Leiomyomatous uterus

endometriosis Explanation Endometriosis is the growth of tissue outside the uterus. The ectopic endometrial tissue can proliferate, and infiltrate, and spread to remote sites elsewhere in the body. Sites that endometriosis is most often found are, ovarian, pelvic peritoneum, anterior and posterior cul-de-sac, uterosacral, round, and broad ligaments, and fallopian tubes. An ectopic pregnancyis one that develops at any site other than the endometrium. Symptoms of an ectopic pregnancy are abdominal pain, possibly a missed period, abnormal bleeding at the time of presentation is not uncommon. Physical characteristics of an adnexal mass that may give a presumptive diagnosis: consistency (cystic, solid or both), size (in centimeters) morphology and surface contour (smooth, nodular, vague or sharp borders), location around the uterus, mobility (free or fixed to adjacent structures) tenderness, unilateral or bilateral. The structures supporting the urethra (urethrocele), bladder (cystocele), uterus, posterior wall of the vagina (enterocele), and rectum (rectocele) weaken. Often symptoms fo this pelvic relaxation occur at or after menopause because of the lack of hormonal effect, causing atrophy of these tissues. Each of these structures has different presenting symptoms. A leiomyoma is a benign uterine tumor. Other names for leiomyoma include myoma, fibroid, and fibromyoma. Symptoms associated with leiomyoma are bleeding, pressure, and pain.

A 23-year-old Hispanic female delivers a healthy male child by normal vaginal delivery. She had a small episiotomy which was clean cut. She is discharged home after 48 hours but returns to the ER on the fifth post partum day with fever and chills. She says she has had foul-smelling vaginal discharge and lower abdominal pain for the last 24 hours. On exam, she has a temperature of 102° F, BP 110/60 mm of Hg, pulse 118/min, and SPO2 92%. Lungs are clear, and she has no pallor, cyanosis, or icterus. Abdominal exam demonstrates tenderness in the suprapubic area and both lower quadrants. Pelvic exam shows foul smelling purulent vaginal discharge and tender uterus. The episiotomy site is non-tender. Labs show Hb 0f 11g/dl, WBC 13,500, bands 11%, and platelets 350,000/uL. Urinalysis is pending. Question The most likely diagnosis is: Answer Choices 1 Endometritis 2 Pyelonephritis 3 Wound cellulitis 4 Cervicitis 5 Cystitis

endometritis Explanation This female patient is suffering from post partum endometritis as obvious by recent delivery, fever, foul-smelling vaginal discharge, and tender pelvic exam. Other risk factors for endometritis include invasive gynecological procedures, retained products of conception, intrauterine devices, submucosal fibroids, multiple sexual partners, unprotected intercourse with infected partner who may be asymptomatic, etc. Leukocytosis with bandemia supports diagnosis. Blood cultures, urine culture, and cervical culture may be done but treatment should not be delayed. Broad spectrum antibiotics to cover beta lactamase and anaerobic organisms like clindamycin with an aminoglycoside are given. Pyelonephritis is unlikely since she has no back pain or costovertebral angle tenderness, and foul vaginal discharge is absent in pyelonephritis. UA and urine culture should rule it out. Wound infections are rare in a small, clean-cut episiotomy. Fever with foul vaginal discharge does not occur, and the wound area will be obviously erythematous, indurated, and tender. Even in third or fourth degree lacerations antibiotic therapy is usually unnecessary. Cervicitis is either asymptomatic or has purulent vaginal discharge without fever, chills, tender abdominal exam, or leukocytosis. Cystitis may be associated with low grade fever and suprapubic tenderness, but dysuria is predominant and vaginal discharge absent. UA confirms diagnosis.

A 49-year-old woman presents with intermittent vaginal bleeding and anemia for 3 months. After her endometrial biopsy, you confirm that she has endometrial carcinoma. What is the largest risk factor for endometrial cancer? Answer Choices 1 Estrogen exposure 2 Oral contraceptive pill 3 Polycystic ovary 4 Granulosa-theca cell ovarian tumor 5 Obesity

estrogen exposure Increased risk of endometrial cancer has been associated with the following: Estrogen exposure is the largest risk factor: Sources of estrogen exposure may be endogenous or exogenous. There is a significant correlation between use of oral estrogen and endometrial cancer, when estrogen therapy is administered without the protective effects of cyclic progestin. Obesity: Women who are 20 - 50 lbs. over their ideal body weight have a 3-fold increased risk for endometrial cancer. Polycystic ovary disease: This is the disease risk attributed to stimulation release of unopposed estrogen from the ovaries. Granulosa-theca cell ovarian tumors: A hormonally active estrogen-producing stromal tumor of the ovary.

A 26-year-old woman presents with a 3-month history of amenorrhea. She has been sexually active with her husband, and she has been trying to conceive for the past 1 year. A pregnancy test is obtained, and it is negative. On examination, she appears overweight; there is acne and hirsutism. Her past menstrual history is significant for irregular menstrual cycles; they are not associated with dysmenorrhea or menorrhagia. Question What is the best management for her infertility? Answer Choices 1 Weight loss and exercise 2 Androgen receptor antagonist 3 Metformin 4 Estrogen receptor modifiers 5 InVitro fertilization

estrogen receptor modifiers Explanation Estrogen receptor modifiers is the correct answer. Clomiphene citrate is a commonly used drug in the management of anovulatory infertility. The vignette describes a classic case of polycystic ovarian syndrome, which is the most common cause of anovulatory infertility. It acts centrally by inhibiting the negative feedback of estrogen on the release of gonadotropins. This results in higher levels of follicle stimulating hormone (FSH) and luteinizing hormone (LH), which promotes the development and release of an ovum. Weight loss and exercise is incorrect. They are key to the management of PCOS, and they do so by reducing Insulin resistance and improving glycemic control. However, they do not correct anovulation. Metformin is incorrect. Metformin reduces insulin resistance and improves peripheral glucose uptake while promoting weight loss. However, it does not correct infertility. In vitro fertilization is incorrect. It is a viable option, and it is often used in infertility after medical management has failed. It is expensive, invasive, and time consuming; there is no guarantee of success. Therapy with clomiphene results in fertility in a number of women with PCOS and should be the first line of therapy.

A 20-year-old woman is brought to the emergency room (ER) by the police. She was assaulted and raped 3 hours ago. Her last menstrual period (LMP) was 22 days ago, and her period lasts for 4 - 5 days with a 28-day menstrual cycle. She is not using any contraceptive method. What hormone is mainly produced by the ovary at this point of her menstrual cycle? Answer Choices 1 Estrogen 2 Follicle stimulating hormone (FSH) 3 Inhibins 4 Luteinizing hormone (LH) 5 Progesterone

progesterone Explanation The correct response is progesterone. The 5th day of the menstrual period corresponds to the follicular phase of the cycle, which starts from the first day of bleeding to the day of the LH peak. In this phase, the FSH, which is produced by the pituitary, stimulates the development of follicles in the ovaries, with only 1 follicle dominant at the end. During the second half of this phase, the granulosa cells of the follicle begin to produce estrogen. Ovulation occurs approximately at the middle of the menstrual cycle (day 14). At the end of the follicular phase, a rise in the level of estrogens takes place, preceding the LH peak. The ovulation will succeed by an average of 30 hours after this peak. The 22nd day of the menstrual period corresponds to the luteal phase of the menstrual cycle. The luteal phase starts from the day of ovulation to the first day of the menstrual period. In this phase, the ovarian corpus luteum begins to produce progesterone. If there is fecundation fertilization of the ovum, the production of progesterone continues for 4 to 5 weeks, until the placenta can take over and produce enough of the hormone to maintain pregnancy. If there is no fertilization, progesterone levels decrease and the menstruation occurs. Estrogens are produced by the granulosa cells of the follicle. They are very important in the follicular phase; their level is very low at the beginning of this phase and reaches a maximum point at the middle of the cycle, just before the LH peak. They are the feedback for the LH production and (in part) for the FSH production. Inhibins are also hormones produced by granulosa cells of the follicle, and they are part of the feedback for the FSH. Inhibin B levels rise during the luteal-follicular transition, are highest during the mid follicular phase, then go up again during the LH peak, and finally decrease in the late follicular phase. Inhibins A levels decrease during the late luteal phase.

A 62-year-old obese woman presents due to urine leakage. She has had some symptoms for about 2 years, but they are getting worse. She leaks urine when she coughs or sneezes. She is not very active, but if she jumps, she also leaks urine. She wears a pad daily, as she has leakage daily. The amount varies from a few drops to a gush. The patient denies hematuria, dysuria, and pelvic pain. She sometimes feels vaginal pressure and fullness. She is considering quitting her job because she is embarrassed to be in public when she has urine leakage. Her past medical history reveals she is menopausal; she has had 4 vaginal deliveries. She has no other known medical conditions; she has not had any surgeries; she takes no medications and has no allergies. She is married and works part-time at a call center; she denies the use of tobacco, alcohol, and drugs. On physical exam, she is obese, with an atrophic vulva/vagina. The pelvic examination reveals downward and forward rotation of the vaginal wall, with an anterior bulging when the patient is asked to strain. The remainder of her exam is normal. A dipstick urinalysis is normal. What is the most appropriate intervention for this patient's current condition? Answer Choices 1 Antibiotic therapy 2 Daily cranberry juice or supplement 3 Start prescription oxybutynin 4 Surgical repair 5 Use over-the-counter phenazopyridine (Azo) as needed

surgical repair Explanation This patient is presenting with a cystocele, a herniation of the bladder wall into the vagina. Common symptoms may include stress urinary incontinence (SUI) and a feeling of vaginal fullness; other symptoms include incomplete voiding and dyspareunia. Of the choices listed, surgical repair is the most appropriate intervention. Other possible approaches to cystocele treatment include pelvic floor physical therapy and use of a pessary. The type of surgical approach to cystocele varies, from a less-invasive mesh sling to a more invasive colpopexy. Antibiotic therapy would be appropriate if this patient's urinary symptoms were due to a cystitis, or urinary tract infection (UTI). Common signs and symptoms of a UTI include urinary frequency and/or hesitancy, dysuria, and (possibly) gross hematuria; urinalysis shows white blood cells, red blood, cells and nitrites. Some women have both a cystocele and a UTI, so it is important to rule out infection when evaluating urinary tract problems. Daily cranberry juice and/or supplements are often used for urinary tract health. However, limited evidence supports cranberry's benefits related to UTI prevention. There is no evidence that cranberry juice or supplements is helpful for cystoceles and stress urinary incontinence. If this patient were suffering overactive bladder (detrusor instability) with resulting urge incontinence, it would be helpful to start prescription oxybutynin, an anticholinergic medication. However, this patient's problem is primarily anatomic, and most medications are ineffective in treating cystocele. If this patient was experiencing both stress and urge incontinence (mixed incontinence), the medication (along with surgery) would be indicated. Patients with frequent UTI may be instructed to use over-the-counter phenazopyridine (Azo) as needed. The phenazopyridine helps with bladder pain and has mild bacteriostatic properties. However, it does not have a role in cystocele and SUI. This medication also turns the urine bright orange, and it can interfere with urinalysis dipstick interpretation.

A 25-year-old woman presents with severe abdominal pain for the last few hours. The patient admits to pelvic pain and vagina discharge for over 1 month, but she refused to make an appointment with her provider to be evaluated. She last had unprotected intercourse with an ex-boyfriend 3 months ago. Her examination reveals a fever of 103oF and abdominally rigidity, with light palpation that is worst in the left lower quadrant. She does not tolerate a speculum exam. A pelvic ultrasound reveals a 4 x 5 cm left adnexal mass. Urine hCG is negative, WBC is elevated, with a left shift. What is the most likely diagnosis? Answer Choices 1 Appendicitis 2 Tubo-ovarian abscess 3 Ovarian cancer 4 Ectopic pregnancy 5 Diverticulitis

tubo-ovarian abscess Explanation This patient present with an acute abdomen that was preceded by pelvic pain and high-risk intercourse. Her history, physical examination, and diagnostic findings are all consistent with a tubo-ovarian abscess. In cases of appendicitis and diverticulitis, you would not expect to find an adnexal mass; the age and presentation of the patient are not consistent with ovarian cancer, and her negative hCG rules out pregnancy, including ectopic pregnancy.

A 20-year-old college football player presents with a chief complaint of a dull ache in his scrotum after prolonged standing on the sideline. It seems to get worse with vigorous activity and is relieved by lying down. Dilated veins in the left scrotum are observed on inspection, and both testicles are palpable and without masses. What is the most likely diagnosis? A varicocele B spermatocele C hydrocele D testicular mass

varicocele A A varicocele can be recognized by the presence of scrotal enlargement caused by dilation of the pampiniform venous plexus. Varicoceles present as a "bag of worms" in the spermatic cord and are more prominent when the patient stands. More than 80% of the time, varicoceles occur on the left side. Hydroceles and spermatoceles are caused by fluid collection and are usually asymptomatic. Testicular masses must always be included in the differential diagnosis of scrotal masses, as they generally present as painless.


Ensembles d'études connexes

Chapter 2 Basic Cost Accounting Concepts

View Set

Human Resource Management Exam 3

View Set

ap bio quiz 6.5-6.6 gene expression

View Set

Abraham Lincoln and Robert E. Lee's writings

View Set

The Name Jar by Yangsook Choi Review Questions

View Set

Ch 12: Reports on Audited Financial Statements

View Set

Foundations for Living A 2020 Unit 1

View Set